PT 32 Game # 2 Forum

Prepare for the LSAT or discuss it with others in this forum.
Post Reply
roballen

New
Posts: 55
Joined: Wed Jun 23, 2010 9:34 am

PT 32 Game # 2

Post by roballen » Tue Aug 24, 2010 9:11 pm

Can someone please show me how they completed this game? Also, I had a hard time with the following rule, if someone can please explain: At least as many French novels as Russian novels are selected

JJDancer

Gold
Posts: 1564
Joined: Sun Jul 26, 2009 7:41 pm

Re: PT 32 Game # 2

Post by JJDancer » Tue Aug 24, 2010 11:16 pm

Either 5 or 6 out of 9 novels/plays will be selected.
There are 3 french novels, and 3 russian novels. 2 french plays, and 1 russian play.

A) No more than 4 french works (could be combo of novels and plays). Don't have to select 4. 4 is the MAX.
B) Either 3 or 4 novels are selected.
C) However many russian novels are selected, at least that many French novels will be selected too. There can be more french novels selected than russian novels but NOT LESS.
Example: you can select 2 russian novels and 2 french novels.
you can select 2 russian novels and 3 french novels.
you CAN'T select 3 russian novels and 2 french novels.
D) If 2frenchplays --> Not russianplay
Contrapositive: If russianplay selected --> 2 french plays are NOT selected. (could select 1 french play, but not both)

JJDancer

Gold
Posts: 1564
Joined: Sun Jul 26, 2009 7:41 pm

Re: PT 32 Game # 2

Post by JJDancer » Tue Aug 24, 2010 11:21 pm

7. Answer is C because it doesn't violate any rules.
How I did it: POE. Pick a rule and check each answer choice against it. Keep moving to the next rule until you are left with one good answer.
A violates the french at least as many as russian novel
B violates if both french plays then no russian play
D violates only having 4 french works - it has 5
E violates having only 3 or 4 novels - it has 5

JJDancer

Gold
Posts: 1564
Joined: Sun Jul 26, 2009 7:41 pm

Re: PT 32 Game # 2

Post by JJDancer » Tue Aug 24, 2010 11:30 pm

8. COULD BE TRUE question - if the answer can be made to work within the parameters of the rules, pick it and move on (assuming you REALLY understand the rules). Only one of the answers CAN be true, all others MUST be false.

D and E are clearly wrong
Can't have 5 french works (limit is 4)
Can't have 6 novels (If 3 russian are selected -- at least 3 french novels must be too). Max is 4 novels. Min is 3.

So let's check out A B and C
C is also easy to eliminate because if you select both french plays, cant select the russian play. Thus, you can't select all 3 plays

A or B?
A: create a hypothetical without russian novels
Fn, Fn, Fn, Fp, Rp
IT WORKS! I have 5 out of 9 works used
3 novels
No more than 4 french works

B- just to check it:
Fn, Rn, Fp, Fp,
If I only have 1 Fn then I can't have 2 Rn because Fn is equal to or greater than Rn.
If I use both Fp, then I can't use Rp
I end up with only 4 works selected. I need either 5 or 6

JJDancer

Gold
Posts: 1564
Joined: Sun Jul 26, 2009 7:41 pm

Re: PT 32 Game # 2

Post by JJDancer » Tue Aug 24, 2010 11:35 pm

9. Asks for a complete and accurate list. Meaning all the choices MUST be correct (work w the premise given which is 3 Fn)
And the list must be complete, meaning there should be enough selections and everyone that must go together is present.

A - this would only give us 4 total works -- Need 5 or 6
B - Gives us 5 french works total - max is 4
C - ANSWER
D - Gives us 5 french works total - max is 4
E - Gives us 5 novels total - max is 4

Want to continue reading?

Register now to search topics and post comments!

Absolutely FREE!


User avatar
3|ink

Platinum
Posts: 7393
Joined: Wed Dec 16, 2009 5:23 pm

Re: PT 32 Game # 2

Post by 3|ink » Tue Aug 24, 2010 11:45 pm

10. If less than 2 french novels are selected, (for instance, 1), then only one Russian novel can be selected. This means there are less than 3 novels. It also means that only 4 works can be selected at most, since the 2 french plays can go with the russian play. The rule requires at least 5 works.


Answer D is correct.

JJDancer

Gold
Posts: 1564
Joined: Sun Jul 26, 2009 7:41 pm

Re: PT 32 Game # 2

Post by JJDancer » Tue Aug 24, 2010 11:49 pm

3 ink you beat me to it :)
Thanks.

Finally, 11.
You are looking for which one can't be true.
Answer: A
Reason: Even if you select all 3 Fn, without any Rn, you would need TWO plays
A only has one play -- giving you only 4 works selected (max)

Want to continue reading?

Register for access!

Did I mention it was FREE ?


Post Reply

Return to “LSAT Prep and Discussion Forum”